Understanding the Integration of Inverse Trigonometric Functions

  • Thread starter Thread starter escryan
  • Start date Start date
  • Tags Tags
    Integration
Click For Summary
The integral of 1/(1+x^2) dx equals arctan(x) due to the relationship between the tangent function and its inverse. This can be understood through implicit differentiation, where y = tan^(-1)(x) leads to y' = 1/(sec^2(y)). The integral cannot be solved using natural logarithm rules or simple substitution because it involves trigonometric identities, specifically that 1 + x^2 equals sec^2(x). Recognizing these identities is essential for understanding the integration process, and further exploration of trigonometric substitutions will clarify the concept.
escryan
Messages
13
Reaction score
0
I'm just curious as to how

<br /> \int (\frac{1}{1+x^{2}}) dx<br />

comes to be

<br /> \tan^{-1} (x)<br />


I was able to find the formula on a table of integrals, but I'd just like to know why it works like this, and why we can't use a natural log rule or a substitution method to find this out.

Thanks for reading!
 
Physics news on Phys.org
y=\tan^{-1}x

x=\tan y (Implicit Differentiation)

y&#039;=\frac{1}{\sec^2 y}

Substitute with a trig identity and you will see why it's true.
 
Last edited:
You should surely be able to see why using u-substitution with the natural log will not work. My guess is that you are just given that anything in that form is going to be arctan and you will learn how to do this later on with Trigonometric Substitutions.

Remember

<br /> Sin^2(x) + Cos^2(x) = 1<br />

and from that

<br /> tan^2(x) + 1 = Sec^2(x)<br />

You can these use these identities when you notice that 1+x^2 in

<br /> \int \frac{1}{1+x^2}<br />

can be substituted as sec^2(x)

there is obviously more to it than that, but to get you started.
 
Last edited:
Question: A clock's minute hand has length 4 and its hour hand has length 3. What is the distance between the tips at the moment when it is increasing most rapidly?(Putnam Exam Question) Answer: Making assumption that both the hands moves at constant angular velocities, the answer is ## \sqrt{7} .## But don't you think this assumption is somewhat doubtful and wrong?

Similar threads

Replies
3
Views
2K
  • · Replies 105 ·
4
Replies
105
Views
6K
  • · Replies 22 ·
Replies
22
Views
3K
  • · Replies 19 ·
Replies
19
Views
2K
Replies
19
Views
2K
  • · Replies 6 ·
Replies
6
Views
2K
Replies
5
Views
1K
  • · Replies 14 ·
Replies
14
Views
3K
  • · Replies 12 ·
Replies
12
Views
2K
Replies
7
Views
2K